Help finding the behaviour of this series?

  • Thread starter Thread starter Dell
  • Start date Start date
  • Tags Tags
    Series
Click For Summary
The discussion focuses on determining the convergence or divergence of the series 0 + ln(2)/4 + ln(3)/9 + ln(4)/16 + ... + ln(n)/n^2. The user employs comparison tests with a known series B(n) to establish convergence criteria but struggles to find an appropriate series for comparison. It is noted that series involving logarithms can be challenging due to the comparative behavior of logarithmic and polynomial functions. A suggestion is made to graph ln(x) alongside various polynomial functions to better understand their growth rates. This graphical approach may assist in identifying a suitable convergent polynomial series that exceeds the original series.
Dell
Messages
555
Reaction score
0
given the following series and i am asked if it converges or diverges in its boundaries

0 + ln(2)/4 + ln(3)/9 + ln(4)/16 + ... + ln(n)/n^2

to find out if series converge/diverge, i have been using comparisons, finding a similar series, B(n) whose behaviour i know or can easily find, then saying :
# if B(n) > A(n), and B(n) converges, then A(n) converges
# if B(n) < A(n), and B(n) diverges, then A(n) diverges
# if A(n)/B(n)=K (K= not 0, not ∞) then A(n) and B(n) behave the same

but i cannot find any series that answers to any of these 3 condidtions, any ideas? the series must either have a known or relatively simple to find behaviour.
 
Physics news on Phys.org
Finding the convergence of series involving logs can be tough because it can be hard to see how log behaves in comparison to polynomials. To get a good sense of this, try graphing in the same window y=ln(x) along with some polynomials such as y=x, y=x^(1/2), y=x^(1/5), etc. What you should generally find is that all x^a (with a > 0) is less than ln(x) for large x. This should help you find a convergent polynomial series greater than yours.
 
Question: A clock's minute hand has length 4 and its hour hand has length 3. What is the distance between the tips at the moment when it is increasing most rapidly?(Putnam Exam Question) Answer: Making assumption that both the hands moves at constant angular velocities, the answer is ## \sqrt{7} .## But don't you think this assumption is somewhat doubtful and wrong?

Similar threads

  • · Replies 3 ·
Replies
3
Views
1K
  • · Replies 2 ·
Replies
2
Views
2K
Replies
8
Views
2K
  • · Replies 17 ·
Replies
17
Views
2K
  • · Replies 1 ·
Replies
1
Views
2K
  • · Replies 7 ·
Replies
7
Views
2K
  • · Replies 4 ·
Replies
4
Views
1K
Replies
5
Views
2K
  • · Replies 14 ·
Replies
14
Views
2K
Replies
7
Views
2K